Lorentz-Transformation des Vakuumzustands

Im Allgemeinen der Hamiltonian H hat einen Vakuumerwartungswert ungleich Null (VEV):

H | Ω = E 0 | Ω ,
Wo | Ω ist der Vakuumzustand. Die Nullpunktsenergie des Quantenoszillators ist ein gutes Beispiel für ein nicht verschwindendes VEV.

Das Problem fällt mir auf, wie wird dieser Vakuumzustand bei der Lorentz-Transformation umgewandelt?

Einerseits können wir uns vorstellen, dass dieser Vakuumzustand die triviale Darstellung der Poincare-Gruppe ist, die eine Lorentz-Invariante ist. ( In Wigners Klassifikation ist der Vakuumzustand im ( 0 , 0 , 0 , 0 ) Klasse, wenn ich das richtig verstehe.) Also

U ( Λ ) | Ω = e ich θ | Ω

Andererseits hat der Vakuumzustand einen 4-Impuls ungleich Null: P = ( E 0 , 0 ) . Es scheint also, dass es sich wie andere Irreps transformieren sollte:

U ( Λ ) | P , σ = σ ' C σ ' σ | Λ P , σ ' ,
Nun, das Wichtigste ist, Λ P könnte einen 3-Impuls ungleich Null haben. Aber der Vakuumzustand sollte null 3-Impuls haben. Knall!

In Lehrbüchern besteht ein gängiges Verfahren zur Überwindung dieses Widerspruchs darin, den Hamilton-Operator zu verschieben H H E 0 ; aber die Poincare-Algebra wäre betroffen:

[ K ich , P J ] = δ ich J H [ K ich , P J ] = δ ich J H + δ ich J E 0

Danke schön.

Das sind viel zu viele Fragen auf einmal. Könntest du vielleicht die Frage fokussieren. Sie könnten einige davon auch als andere SE-Fragen stellen.
Entschuldigung, ich verstehe das nicht. In einer relativistischen QFT wird angenommen, dass der Vakuumzustand die einzige Zustandsinvariante (also ein Grundzustand) unter der gesamten Poincaré-Gruppe ist und folglich einen Null-Vier-Impuls hat.
@V.Moretti: Aber das Vakuum ist auch eine Darstellung der Poincare-Gruppe, die sich nach der Regel der Poincare-Gruppe transformieren sollte. Wenn es also in einem Rahmen einen nicht verschwindenden 4-Impuls hat, kann es in einem anderen Rahmen einen nicht verschwindenden 3-Impuls erlangen, was der Lorentz-Invarianz widerspricht.
Der harmonische Oszillator ist kein gutes Beispiel: Er ist nicht relativistisch und nicht translationsinvariant.
@ Adam OK. Was ist mit der Freifeldtheorie, die als Überlagerung von Quantenoszillatoren angesehen wird?
Es gibt zwei Begriffe der Poincaré-Invarianz: (1) im Hilbert-Raum der Theorie existiert eine einheitliche Darstellung der Poincaré-Gruppe, die die Algebra der Observablen kovariant transformiert. (2) Zusätzlich zu (1) gibt es auch einen eindeutigen Zustand, der unter der Wiederholung 1 unveränderlich ist. In der QFT werden (1) und (2) beide angenommen.
Ein freies Quantenfeld ist keine Überlagerung von Oszillatoren. Dieses Bild ist nur heuristisch, es ist eine Fock-Darstellung einer geeigneten Weyl-Algebra.
Das Vakuum in der relativistischen QFT ist die triviale Repräsentation der Poincaré-Gruppe, also hat es kein vierfaches Momentum. Um es tatsächlich zu beweisen, reicht es aus, dass der Zustand unter vier Übersetzungen invariant ist (und das wird angenommen).
@V.Moretti: Ich denke, der Punkt des OP ist, dass Sie, wenn Sie mit einer freien Theorie beginnen, diese Theorie im Impulsraum diagonalisieren können, wo Sie nur eine Sammlung harmonischer Oszillatoren haben. Dann stellen Sie fest, dass das Vakuum eine Nullpunktsenergie hat k ω k / 2 . (Das ist die Vakuumenergie, die den berühmten Fehler von 120 Größenordnungen für die „Vorhersage“ der kosmologischen Konstante verursacht.)
@V.Moretti: Was ist mit dem Grundzustand einer Wechselwirkungstheorie? Geht man davon aus, dass es auch unter Translation invariant ist oder lässt es sich irgendwie beweisen? Siehe meine Frage: physical.stackexchange.com/questions/99920/…
Sie wird auch für interagierende Theorien angenommen.

Antworten (1)

Wie in den Kommentaren erwähnt wurde, wird angenommen, dass die QFT einen Vakuumzustand hat, der vernichtet wird P μ . Dies ist tatsächlich ein sehr wichtiger Punkt, da es einer der entscheidenden Unterschiede zwischen der QFT im flachen Raum und der QFT in der gekrümmten Raumzeit ist. Dies wird in Walds Buch QFT in Curved spacetime erklärt. Im Wesentlichen wird in Quantentheorien die "kinematische Struktur" durch die kanonischen Vertauschungsbeziehungen zwischen dem kanonischen Ort und Impulsen (Feldern und konjugierten Impulsen in der QFT) festgelegt. In der nichtrelativistischen QM gibt es eine endliche Anzahl von Freiheitsgraden, und daher sagt Ihnen das Stone-von-Neumann-Theorem, dass es einen eindeutigen Hilbert-Raum und eine Auswahl von Operatoren auf dem Hilbert-Raum gibt. Der Satz gilt nicht für unendlich viele dof (QFT), und es gibt tatsächlich unendlich viele inäquivalente Möglichkeiten für den Hilbert-Raum für Quantenfelder. In der flachen Raumzeit gilt die Forderung, dass es einen solchen Zustand gibt P μ | 0 = 0 wählt einen einzigartigen Hilbert-Raum aus. In gekrümmten Raumzeiten, die im Allgemeinen keine Tötungsvektoren haben, ist es unklar, wie man einen eindeutigen Hilbert-Raum ausfindig macht, und dies ist eine große Schwierigkeit bei der QFT im gekrümmten Raum.

Die Antwort ist also, dass es angenommen wird P μ | Ω = 0 und das ist eine sehr wichtige Annahme.

Obwohl das von Ihnen erwähnte Subtraktionsverfahren in vielen Lehrbüchern verwendet wird, halte ich es nicht für wirklich die richtige Lösung. Das Argument ist normalerweise, dass der Subtraktionsterm nicht beobachtbar ist, da er nur eine Phase zur s-Matrix beiträgt und wir nur Energieunterschiede messen, aber das gilt wirklich nicht, wenn Schwerkraft oder Supersymmetrie im Bild sind. Tatsächlich ist es, wie Sie bemerken, nicht klar, dass die Poincare-Algebra unter einer solchen Subtraktion schließt.

Die Operatoren in zwei Theorien mit unterschiedlichen Hamiltonoperatoren müssen nicht gleich sein, sie müssen sich nur so ändern, dass die Poincare-Algebra immer noch schließt. Wenn Sie einen originalen Hamiltonian hätten H 0 und du hast es gestört H 0 H = H 0 + v , dann senden H H E 0 läuft im Grunde darauf hinaus, den Hamiltonian zu stören H 0 von v E 0 . Für Störungen, die Lorentz-invariant aus lokalen Feldern aufgebaut sind, ist es immer möglich, die Algebra so zu verändern, dass sogar in der Wechselwirkungstheorie die Algebra schließt.

Tatsächlich ist dies im Grunde die Antwort auf die Frage: Welche Arten von Störungen kann ich zum Hamilton-Operator hinzufügen, die zu einer Lorentz-Invariantentheorie führen, die dem Cluster-Zerlegungsprinzip gehorcht? Ich werde skizzieren, wie das Argument geht.

In der freien Theorie haben wir Zustände der Form A P . . . | 0 . Dies ist die übliche Definition unseres freien Hilbert-Raums. In Betracht ziehen [ K ich 0 , H 0 ] = ich P ich . Wir wollen, dass das Momentum-Label nach der Störung dasselbe bedeutet (wir haben nur unsere Energien verändert), aber wir haben gesendet H 0 H 0 + v . So naiv greifen wir einen Begriff auf [ K ich 0 , v ] . Von diesen beiden Betreibern zu verlangen, dass sie pendeln, ist eine zu starke Einschränkung. Also lassen wir stattdessen zu K ich 0 K ich 0 + K ich v so dass

[ K 0 , v ] + [ K v , H 0 ] + [ K v , v ] = 0
Verwendung der Kommutierungsbeziehungen [ A , A ] , etc Sie können zeigen, dass, wenn V von der Form ist
v = D 3 X v ( ϕ ( X ) )
Wo ϕ ( X ) ist ein lokales Feld, das aus den Erstellungs- und Vernichtungsoperatoren aufgebaut ist, dann eine Lösung
K ich v = D 3 X X ich v ( ϕ )
schließt die Algebra (Sie müssen wirklich nur den von Ihnen erwähnten Kommutator überprüfen). Das soll nicht heißen, dass lokale QFTs die einzige Möglichkeit sind, Lorentz-invariante Quantensysteme zu bauen, es sagt nur, dass lokale QFTs einige der Lösungen sind.

Das Problem ist, dass E 0 ist nicht aus lokalen Feldern aufgebaut und kann nicht einmal wirklich lokal in einer unendlichen Volumen-QFT geschrieben werden. Sie würden etwas von der Form wollen v = D 3 X E 0 v . Für eine solche Störung K v = 0 und wir können nicht zufrieden stellen

[ K ich v , P J ] = δ ich J E 0
Dies ist nicht so überraschend, da eine Konstante zu hinzugefügt wird P 0 aber nicht P ich ist nicht wirklich Lorentz-invariant.

Also meiner Meinung nach ist die Subtraktion eher ein Cheat, der zufällig funktioniert, da wir immer nur wirklich rechnen | Ö u T | S | ich N | 2 wofür es egal ist.

zwei Anmerkungen: 1. Ihre Analyse basiert rein auf pQFT. Gibt es eine Möglichkeit, auf eine breitere Klasse von QFTs zu verallgemeinern? (Ich weiß, dass argumentiert wurde, dass pQFT vielleicht die einzige QFT ist, die Sinn macht.) 2. Gemäß der Quantenmechanik (QM) ist eine einheitliche projektive Darstellung erlaubt. Der Weg, um eine projektive Darstellung aus einer linearen Darstellung zu erhalten, besteht darin, die Algebra um eine zentrale Ladungserweiterung zu verschieben (und Superauswahlregeln aufzuerlegen, wenn die Topologie der Lie-Gruppe nicht trivial ist). Wie wirkt sich dieser Operator auf Ihr Argument aus?
Projektive Darstellungen können aus zentralen Erweiterungen oder nicht-trivialer Topologie der Gruppenmannigfaltigkeit entstehen. Im Fall der Poincare-Gruppe können Sie zeigen, dass die projektiven Darstellungen durch Berücksichtigung der universellen Überdeckung erhalten werden, keine zentrale Erweiterung zulässig ist (dies wird in Weinberg getan). Wenn Sie mit pQFT die perturbative QFT meinen, dann ja, meine Analyse basiert ausschließlich auf pQFT. Ich kenne nicht wirklich viel nicht-perturbatives QFT, an dem SUSY nicht beteiligt ist, also kann vielleicht jemand anderes darauf antworten.